Cartesian Integral to Polar Integral

In summary: So, when we do an integral, we're really doing a sum over all the little annular sectors. In summary, the polar integral is equivalent to the Cartesian integral, and the bounds for the first integral are -5 to 5 and the bounds for the second integral are 0 to \sqrt{ 25-x^{2}}.
  • #1
Abide
16
0

Homework Statement


Change the Cartesian integral to an equivalent polar integral and evaluate

∫∫dydx

The bounds of the first integral (The outermost) are -5 to 5, and the bounds of the second (inner) are 0 to [itex]\sqrt{ 25-x^{2}}[/itex]

Homework Equations



∫∫dydx == ∫∫r(dr)(d[itex]\Theta[/itex])

[itex]x^{2}[/itex]+[itex]y^{2}[/itex]=[itex]r^{2}[/itex]

x = rCos([itex]\Theta[/itex])

y = rSin([itex]\Theta[/itex])

The Attempt at a Solution



I solved for r by setting y equal to [itex]\sqrt{ 25-x^{2}}[/itex]

after doing this I found r to be 5

I know that typically the next step is to find the bounds for \Theta, but I have no clue as to how to do this, I know how to set up the integral and evaluate it, but I do not know how to determine the bounds for [itex]\Theta[/itex] can anyone please explain that to me?

Thanks for your time.
 
Physics news on Phys.org
  • #2
Draw a picture of the region bounded by ##y=\sqrt{25-x^2}## and use it to figure the ##\theta## limits.
 
  • #3
Ok, so it would be 0 to 5[itex]\Theta[/itex]?
 
Last edited:
  • #4
No. The outer limits must be constant. What does your picture look like?
 
Last edited:
  • #5
Last edited:
  • #6
I wasn't looking for you to show me a picture of what Wolfram gives. I want you do describe the given region in words so I know you know what you are looking at. You need to understand the region to figure out the limits.
 
  • #7
It's an arc in the first quadrant, with a y maximum of 5 and an x maximum of 5. I don't really know what you're asking, I apologize. This is holding me up on every problem I try, I have no clue how you get these values for theta...
 
  • #8
What are the polar coordianates of the point (5, 0) in Cartesian coordinates? What are the polar coordinates of the point (-5, 0) in Cartesian coordinates?
 
  • #9
Abide said:
It's an arc in the first quadrant, with a y maximum of 5 and an x maximum of 5. I don't really know what you're asking, I apologize. This is holding me up on every problem I try, I have no clue how you get these values for theta...

What kind of arc? What is it? An arch of a sine curve? A piece of a parabola? A chunk of an ellipse?
 
  • #10
HallsofIvy said:
What are the polar coordianates of the point (5, 0) in Cartesian coordinates? What are the polar coordinates of the point (-5, 0) in Cartesian coordinates?

Well, for (5,0) the Polar Coordinates (r,theta) would be r = [itex]\sqrt{(5^2)+0^2}[/itex] = 5
then the theta value would be Tan[itex]^{-1}[/itex](0)

or (-5,0) the Polar Coordinates (r,theta) would be r = [itex]\sqrt{(-5^2)+0^2}[/itex] = 5
then the theta value would be Tan[itex]^{-1}[/itex](0)

for (0,5) the Polar Coordinates (r,theta) would be r = [itex]\sqrt{(0^2)+5^2}[/itex] = 5
then the theta value would be Tan[itex]^{-1}[/itex](5/0)
 
Last edited:
  • #11
Abide said:
Well, for (5,0) the Polar Coordinates (r,theta) would be r = [itex]\sqrt{(5^2)+0^2}[/itex] = 5
then the theta value would be Tan[itex]^{-1}[/itex](0)

What angle is that, in radians?

or (-5,0) the Polar Coordinates (r,theta) would be r = [itex]\sqrt{(-5^2)+0^2}[/itex] = 5
then the theta value would be Tan[itex]^{-1}[/itex](0)
The same angle?
for (0,5) the Polar Coordinates (r,theta) would be r = [itex]\sqrt{(0^2)+5^2}[/itex] = 5
then the theta value would be Tan[itex]^{-1}[/itex](5/0)

5/0 is undefined. Anyway, just look at the picture and tell us the angles. Do you understand what polar coordinates measure? And you haven't answered what kind of graph that is.
 
  • #12
I don't really know what you mean by explaining the graph...It looks like an elipse, and I believe that polar coordinate measure real and imaginary parts of a number - That may be completely wrong. I am also fully aware that 5/0 is undefined I was just using the definitions presented in my book.

I'm assuming that the measurement is in radians, I'm very lost with this topic and I apologize
 
Last edited:
  • #13
Ok, so I understand this a lot better now after reading the following link...feeling pretty stupid at the moment...

http://www.mathsisfun.com/polar-cartesian-coordinates.html

I now understand that the measurement is in Degrees after taking Tangeant inverse of y/x.

My question now is, how do we determine the bounds over which to use this formula

For example: from (5,0) to (0,5); or (5,0) to (-5,0) etc
 
  • #14
Abide said:
My question now is, how do we determine the bounds over which to use this formula

For example: from (5,0) to (0,5); or (5,0) to (-5,0) etc
It will help you enormously to acquire a grasp of what's going on rather than just reaching for formulae.
When we do an integral in Cartesian, dxdy, we're breaking the region of interest up into little rectangles. One such rectangle is from x to x+dx in the x-direction and y to y+dy in y-direction. That's fine when you have two parallel straight line boundaries (like, x≥a, x≤b), but can get messy otherwise.
In polar co-ordinates, we break it into little annular sectors, r going from r to r+dr and theta from θ to θ+dθ. (The area of such a sector becomes, in the limit, rdrdθ, so part of the conversion turns dxdy into that.)
If r is the inner integral, that means you are first of all summing these little sectors from the origin out towards the value of r which bounds the area. Think of this as being done separately for each θ, so θ is constant during this summation. So the range for r in this case is... what?
Having done that integral, we know the value for sectors of angular width dθ, running out from the origin. We now have to sum those as theta runs from its minimum value for the region out to its maximum value for the region. Having drawn the region, what are those minimum and maximum angles?
 

What is the Cartesian integral to polar integral transformation?

The Cartesian integral to polar integral transformation is a mathematical process used to convert an integral expressed in terms of Cartesian coordinates (x and y) to an integral expressed in terms of polar coordinates (r and θ).

Why is the Cartesian integral to polar integral transformation useful?

This transformation is useful because it allows for easier integration of functions that are expressed in polar coordinates. It also helps in solving problems that involve circular, symmetrical shapes.

What is the formula for converting a Cartesian integral to a polar integral?

The formula for converting a Cartesian integral to a polar integral is:
∫∫f(x,y)dxdy = ∫∫f(rcosθ, rsinθ)rdrdθ

What are the limits of integration when converting from Cartesian to polar coordinates?

The limits of integration for the radial variable (r) are determined by the distance from the origin to the outermost boundary of the region of integration. The limits for the angular variable (θ) are typically 0 to 2π, representing a full circle.

What are some common applications of the Cartesian integral to polar integral transformation?

Some common applications of this transformation include calculating areas and volumes of circular or symmetrical shapes, solving problems in physics and engineering involving polar coordinates, and simplifying complex integrals in calculus.

Similar threads

  • Calculus and Beyond Homework Help
Replies
3
Views
248
  • Calculus and Beyond Homework Help
Replies
7
Views
971
  • Calculus and Beyond Homework Help
Replies
5
Views
1K
  • Calculus and Beyond Homework Help
Replies
6
Views
940
  • Calculus and Beyond Homework Help
Replies
11
Views
2K
  • Calculus and Beyond Homework Help
Replies
4
Views
109
  • Calculus and Beyond Homework Help
Replies
2
Views
1K
  • Calculus and Beyond Homework Help
Replies
3
Views
1K
  • Calculus and Beyond Homework Help
Replies
27
Views
2K
  • Calculus and Beyond Homework Help
Replies
5
Views
670
Back
Top